gekrümmter Leiter exakter Widerstand ?

Hallo zusammen, ich habe Probleme einen L=F6sungsansatz f=FCr die Berechung des Widerstands eines gerk=FCmmten Leiters zu finden.

Wenn man z.B. einen einen Leiter hat, der im Querschnitt ein Dreieck ist und um symetrischen Halbkreis um eine Achse bildet. Wir w=FCrdet ihr davon den genauen Widerstand berechnen?

Bild der Aufgabe:

formatting link
Bin f=FCr Hilfe sehr dankbar !

Viele Gr=FC=DFe aus K=F6ln

Daniel Hemmerling

P=2ES: Das ist jetzt keine Hausaufgabe oder so, leider hat uns der Prof davon keine L=F6sung gegeben und ich w=FCrde die Frage auch nicht stellen, wenn ich jemanden Fragen k=F6nnte ;)

Reply to
Daniel H
Loading thread data ...

Ich würde den Leiter mal auf viele dünne, dreieckige Litzendrähte zerfieseln und damit rumrechnen. Vielleicht kommt dann die Erleuchtung.

Servas

Reply to
Franz Glaser (KN)

Genau, das ist der praktische Ansatz (k == Kappa):

dk(r)=k * g/h * (r-R)^2 für R

Reply to
Stefan Ollermann

X-No-Archive: Yes

begin quoting, Daniel H schrieb:

Wie liegen die Äquipotentialflächen im Leiter?

Hübsch!

^ Du plenkst.

Ich komme auf R = 2,873 µOhm und eine Stromdichte, die umgekehrt proportional zum Zentrumsabstand ist - letzteres ist unabhängig vom speziellen Leiterquerschnitt, sondern liegt einfach an der Rotationssymmetrie.

Gruß aus Bremen Ralf

Reply to
Ralf Kusmierz

Danke f=FCr den konkreten Wert. Sorry, aber wieso plenke ich?

Reply to
Daniel H

"Daniel H" schrieb im Newsbeitrag news: snipped-for-privacy@k79g2000hse.googlegroups.com... Hallo zusammen, ich habe Probleme einen Lösungsansatz für die Berechung des Widerstands eines gerkümmten Leiters zu finden.

Wenn man z.B. einen einen Leiter hat, der im Querschnitt ein Dreieck ist und um symetrischen Halbkreis um eine Achse bildet. Wir würdet ihr davon den genauen Widerstand berechnen?

Bild der Aufgabe:

formatting link
Bin für Hilfe sehr dankbar !

Viele Grüße aus Köln

Daniel Hemmerling

P.S: Das ist jetzt keine Hausaufgabe oder so, leider hat uns der Prof davon keine Lösung gegeben und ich würde die Frage auch nicht stellen, wenn ich jemanden Fragen könnte ;)

Hallo Daniel,

teile die Stirnfläche in beliebig dünne vertikale Streifen der Breite (dx).

Fläche: A'=g/h*x*dx x=0 bis x=h

Leitwert eines Streifens.

G'=k*A'/L

Länge L L = (R+x)*pi

Der Gesamtleitwert ist die Summe der Leitwerte dieser schmalen Streifen der Länge (R+x)*pi.

G = Integral G'dx G = Integral k*g/h/pi*x/(R+x))dx von 0 bis h

G = k*g/(h*pi)*(h+R*log(R/(R+h)))

---------------------------------

Auf die Einheiten mm^2 achten.

G = 3.48e5 Siemens

R = 1/G = 2.873e-6 Ohm

Gruß Helmut

Das Integral x/(R+x)dx gibts hier.

formatting link

Reply to
Helmut Sennewald

X-No-Archive: Yes

begin quoting, Daniel H schrieb:

formatting link

Gruß aus Bremen Ralf

Reply to
Ralf Kusmierz

Was der Wicki in seiner umfassenden Weisheit nicht weiß, das hat mit der Unterstreichung zu tun. Manche/einige Reader können das _Unterstreichen_ nur dann, wenn _vorn_ _und_ _hinten_ ein _Leerzeichen_ ! steht. Mein _KNode_ _zum_ _Beispiel_ ist so ein _Reader_ , er zwingt zum _Plenken_ .

Servas

Reply to
Franz Glaser (KN)

U29ycnksIGJyw6R1Y2h0ZSBub2NobWFsIGt1cnogZXVyZSBIaWxmZS4KSWNoIGhvZmZlIGlociBi ZW51dHp0IGVpbmVuIE5ld3NyZWFkZXIgbWl0IGVpbmVtIEZlc3RlcgpaZWljaGVuYWJzdGFuZC4K CkvDtm5udCBpaHIgZGVuIEZlaGxlciBzZWhlbj8KCiBoCuKMoCAgICAgICBnCuKOriAgICBrwrfu n4zijq/un4vCt3gK4o6uICAgICAgIGggICAgICAgID0xLjMxMmU0IFMgKG5pY2h0IGRpZSAzLjQ4 ZTUgUyB3aWUgaWhyIHNpZSBoYWJ0KQrijq4gIO6fjOKOr+KOr+KOr+KOr+KOr+KOr+KOr+KOr+KO r+6fiyBkeArijKEgICAociArIHgpwrdwaQogMAoKbWl0CiAgICAgZwphIOKJlCDun4zijq/un4vC t3gKICAgICBoCgogICAgICAgIC0yCmgg4omUIDXCtzEwCgogICAgICAgICAtMgpnMiDiiZQgM8K3 MTAKCiAgICAga8K3YQpnIOKJlCDun4zijq/ijq/ijq/un4sKICAgICAgbAoKbCDiiZQgKHIgKyB4 KcK3cGkKCiAgICAgICAgICAgNgprIOKJlCA1Ni44wrcxMAoKTWl0IGZyZXVuZGxpY2hlbiBHcsO8 w59lbgoKRGFuaWVsIEhlbW1lcmxpbmcK

Reply to
Daniel H

Hallo, Daniel,

Du (recapture) meintest am 21.09.07:

Bisher kenne ich das Fragezeichen nur als neue Währungseinheit. Dass es auch schon in Formeln auftaucht ...

Viele Gruesse! Helmut

Reply to
Helmut Hullen

SWNoIGthbm4gbWlyIG5pY2h0IGhlbGZlbiwgaXJnZW5kd2FzIG1hY2ggaWNoIGZhbHNjaCwgdmll bGxlaWNodCBrw7ZubnQKaWhyIGphIG1hbCBlaW5lbiBCbGljayBkcsO8YmVyIHdlcmZlbi4KCgog aArijKAgICAgICAgZwrijq4gICAga8K37p+M4o6v7p+Lwrd4CuKOriAgICAgICBoICAgICAgICA9 IDEuMzEyZTQgUyAoaWhyIGhhYnQgMy40OGU1IFMpCuKOriAg7p+M4o6v4o6v4o6v4o6v4o6v4o6v

4o6v4o6v4o6v7p+LIGR4CuKMoSAgIChyICsgeCnCt+6fggogMAoKbWl0CiAgICAgZzIKYSDiiZQg 7p+M4o6v4o6v7p+Lwrd4CiAgICAgIGgKCiAgICAgICAgICAgNgprIOKJlCA1Ni44wrcxMAoKciDi iZQgMQogICAgICAgIC0yCmgg4omUIDXCtzEwCgogICAgICAgIC0yCmcg4omUIDPCtzEwCgpLYW5u IGljaCBkYXMgbmFjaGhlciBhdWYgamVkZW4gZXJkZW5rbGljaGUgRm9ybSBhbndlbmRlbj8KS3Jl aXMsIFZpZXJlY2t0LCBFbGxpcHNvaWQsIGV0YwoKRGFuaWVsIEhlbW1lcmxpbmcK
Reply to
Daniel H

Am 21.09.2007 17:23:04 schrieb Daniel H:

Tja in diesem Posting sind deine Formeln auch lesbar. Vorher war dein Posting total zerstört bzw. viele Sonderzeichen flasch dargestellt.

lG Ferenc

Reply to
Ferenc Staedter

"Daniel H" schrieb im Newsbeitrag news: snipped-for-privacy@k79g2000hse.googlegroups.com...

Hallo Daniel,

Zeige mir doch mal den Lösungsweg deines Integrals und deine Ergebnisformel.

Gruß Helmut

Reply to
Helmut Sennewald

snipped-for-privacy@k79g2000hse.googlegroups.com...

So, habe alles noch mal fein zusammen gefasst ...

Hier die Aufgabe

formatting link

Das ist gegeben:

6 k =3D 56.8=C2=B710

-2 h =3D 5=C2=B710 m

-2 g2 =3D 3=C2=B710 m

F=C3=BCr die L=C3=A4nge:

l =3D (r + x)=C2=B7pi

F=C3=BCr die Fl=C3=A4che:

g a =3D --- r h

Leitwert:

k=C2=B7a g =3D ----- l

Also alles einf=C3=BCgt:

h =E2=8C=A0 g | k=C2=B7----=C2=B7x | h =3D1.312e4 S | ------------ dx =E2=8C=A1 (r + x)=C2=B7pi 0

Ihr habt da 3.48e5 Siemens raus, und dich denke mal, dass ich mich verrechnet habe.

Vielen Dank

Daniel

P=2ES. W=C3=A4hre auch toll wenn ihr mit das Integral oder die Herleitung f= =C3=BCr einen runden Leiter geben k=C3=B6nntet.

Reply to
Daniel H

----- Original Message ----- From: "Daniel H" Newsgroups: de.sci.ing.elektrotechnik Sent: Saturday, September 22, 2007 8:37 PM Subject: Re: gekruemmter Leiter exakter Widerstand?

6

-2

-2

g

Hallo Daniel,

Warum nimmst du g2 statt g?

Nochmals von vorne.

Teilen der Stirnfläche in beliebig dünne vertikale Streifen der Breite dx.

Fläche: A'=g/h*x*dx x=0 bis x=h x=0 ist an der Spitze des Dreiecks (=Abstand R von der Mitte)

Leitwert eines Streifens.

G'=k*A'/L

Länge L L = (R+x)*pi

Der Gesamtleitwert ist die Summe der Leitwerte dieser schmalen Streifen der Länge (R+x)*pi.

G = Integral G'dx

G = Integral k*g/h/pi*x/(R+x))dx von 0 bis h

G = k*g/h/pi*(x-R*log(R+x)) 0 bis h

G = k*g/h/pi*( h-R*log(R+h) -0+R*log(R+0) )

log(a)-log(b) = log(a/b)

G = k*g/(h*pi)*(h+R*log(R/(R+h)))

---------------------------------

G=56.8 * 0.03/(0.05*pi)*(0.05-0.01*log(0.01/(0.01+0.05))) S*m*m/mm^2

Scilab:

-->56.8*0.03/0.05/%pi*(0.05+0.01*log(0.01/0.06)) -->ans = 0.3480300

*1e6 wegen 1/mm2

G=0.3480300*1e6S

R = 1/G = 2.8733158e-6 Ohm

Frage b:

Stromdichte ist proportional G'/A'

S = const/(R+x)

A' = 2*sqrt(h*x-x*x) Durchmesser ist h L = (R+x)*pi G'= k*A'/L

G = Integral(k*2*sqrt(h*x-x*x)/((R+x)*pi)dx x = 0 bis h

Nimm den Solver von Wolfram Research (siehe unten).

Das war jetzt mein Lösungsansatz für beide Aufgaben. Es wäre interessant zu erfahren ob ich richtig liege. Kennt jemand die garantiert richtige Lösung aus irgendeiner Literaturstelle?

Gruß Helmut

Das Integral x/(R+x)dx gibts hier.

formatting link

Reply to
Helmut Sennewald

"Helmut Sennewald" schrieb im Newsbeitrag news:fd5gt9$o3a$00$ snipped-for-privacy@news.t-online.com...

Tippfehler, - statt + G=56.8 * 0.03/(0.05*pi)*(0.05+0.01*log(0.01/(0.01+0.05))) S*m*m/mm^2

Reply to
Helmut Sennewald

"Helmut Sennewald" schrieb im Newsbeitrag news:fd5gt9$o3a$00$ snipped-for-privacy@news.t-online.com...

Hallo Daniel,

Irgend etwas muss an meinem Lösungsansatz falsch sein, da die Formel für große Werte von R nicht auf die Triviallösung konvergiert.

R>>h

G=k*g*h/(2*pi*R)

Sorry, mein Ansatz muss wohl falsch sein.

Gruß Helmut

Reply to
Helmut Sennewald

Nein Helmut, der ist richtig. Du musst nur den Grenzwert richtig berechnen :)

Von oben (h in die Klammer rueber, damit die Dimensionslos wird):

G = k*g/pi * ( 1 + R/h * log (R/(R+h)) )

Fuer grosse Werte von R/h geht ( 1 + R/h * log (R/(R+h)) ) gegen h/(2*R)

Ausrechnen z.B. durch einsetzen in eine geeignete Reihenentwicklung fuer log().

Beste Gruesse, Andreas

Reply to
Andreas 'Nolly' Arnold

PolyTech Forum website is not affiliated with any of the manufacturers or service providers discussed here. All logos and trade names are the property of their respective owners.